آيا شما مايل به راه افتادن تاپيك ماراتن نامساوي مي باشي


  • مجموع رای دهندگان
    339

zz_torna2

New Member
ارسال ها
300
لایک ها
254
امتیاز
0
پاسخ : ماراتن نامساوي

س1:


داریم:


حالا از روش دلتا گیری اثبات تمام میشه.(البته مشتق هم میشد گرفت)

س2:
راه اول:ینسن
راه دوم:نامساوی توانی

برای سوال قبلی:

از تابع logx استفاده و ینسن بزنید و از اتحاد:


:3:
 
لایک ها MBGO

MBGO

New Member
ارسال ها
247
لایک ها
104
امتیاز
0
پاسخ : ماراتن نامساوي

عالی بود،



شما هم سوال بذارید :)
 

zz_torna2

New Member
ارسال ها
300
لایک ها
254
امتیاز
0
پاسخ : ماراتن نامساوي

من کسر ها رو از بین بردم و به این رسیدم:



کمکی میکنه؟
(اگه نه راهنمایی کنید!)
 

Aref

New Member
ارسال ها
1,262
لایک ها
1,008
امتیاز
0
پاسخ : ماراتن نامساوي

اعداد نامنفی ه؟ :13: این نماد اختراع خودتونه؟
اون نابرابری که ثابت کردید درسته، ولی خب چه ربطی به مساله ی ما داره؟
تازه حکم نوشته شده اینه که ثابت کنید:
 
آخرین ویرایش توسط مدیر

Aria Keshtkar

New Member
ارسال ها
61
لایک ها
44
امتیاز
0
پاسخ : ماراتن نامساوي

در حالت کلَی داریم:
که در نابرابری نخست از نابرابری کوشی شوارتز و نابرابری زیر که این هم در واقع همان کوشی شوارتز هست ، استفاده شده است.
حال تغییر متغیَر زیر را انجام داده و داریم :

هم چنین با تغییر متغیَر زیر داریم :
حال به حلَ مسئله ی اصلی می پردازیم :
پس برای حلَ مسئله باید نشان دهیم :
نتیجه ی فوق صحیح است چون داریم :

که به وضوح داریم :

 
آخرین ویرایش توسط مدیر

mehrdad1st

New Member
ارسال ها
93
لایک ها
40
امتیاز
0
پاسخ : ماراتن نامساوي

میشه سوال بعدی رو خودتون بذارید!!
 

Aria Keshtkar

New Member
ارسال ها
61
لایک ها
44
امتیاز
0
پاسخ : ماراتن نامساوي

حتماً امَا ببخشید اگه آسون هستند.




 

AHZolfaghari

Well-Known Member
ارسال ها
935
لایک ها
1,654
امتیاز
93
پاسخ : ماراتن نامساوي

اعداد a,b,c دلخواه هستند و x,y,z نیز چنین رابطه ای با a,b,c دارند:
حالا اثبات کنید :


توجه کنید که a,b,c الزاما مثبت نیستند
 

AHZolfaghari

Well-Known Member
ارسال ها
935
لایک ها
1,654
امتیاز
93
پاسخ : ماراتن نامساوي

حلشو بذارم ؟؟؟؟؟؟چقدر فعالیت کم شده !!!
 

mahdi math

New Member
ارسال ها
152
لایک ها
61
امتیاز
0
پاسخ : ماراتن نامساوي

اعداد a,b,c دلخواه هستند و x,y,z نیز چنین رابطه ای با a,b,c دارند:
حالا اثبات کنید :


توجه کنید که a,b,c الزاما مثبت نیستند
میشه بگید منظور از زیگما پشت ایکس و وای چیه؟:13:
 

AHZolfaghari

Well-Known Member
ارسال ها
935
لایک ها
1,654
امتیاز
93

AHZolfaghari

Well-Known Member
ارسال ها
935
لایک ها
1,654
امتیاز
93
پاسخ : ماراتن نامساوي

خب یه راهنمایی میذارم برای این سوال . انصافا هم آسون نیست . برای xy هر کدوم از x و y رو بصورت مجموع دو عبارت بنویسید که توش مربع کامل باشه تا وقتی تو هم ضرب میشن بشه از نامساوی کوشی استفاده کرد.
 
آخرین ویرایش توسط مدیر

a$hk@n

New Member
ارسال ها
618
لایک ها
440
امتیاز
0
پاسخ : ماراتن نامساوي

سلام من سوالو الان دیدم حقیقتا زیاد روش فکر نکردم ولی چیزی که به ذهنم اومد رو مینویسم;)


---- دو نوشته به هم متصل شده است ----

سلام من سوالو الان دیدم حقیقتا زیاد روش فکر نکردم ولی چیزی که به ذهنم اومد رو مینویسم;)
بعد ثابت کنیم کنیم کمتر مساوی سیکما آ به توان 2 هست البت شاید کارمون سختر شده باشد نمیدانم!!!
 

AHZolfaghari

Well-Known Member
ارسال ها
935
لایک ها
1,654
امتیاز
93
پاسخ : ماراتن نامساوي

سلام من سوالو الان دیدم حقیقتا زیاد روش فکر نکردم ولی چیزی که به ذهنم اومد رو مینویسم;)


---- دو نوشته به هم متصل شده است ----


بعد ثابت کنیم کنیم کمتر مساوی سیکما آ به توان 2 هست البت شاید کارمون سختر شده باشد نمیدانم!!!
یه چیزی رو میگم من . نگاه کنید ! اون کوشی که شما زدید برای تمامی x,y,z ها صادق هستش اما اینجا x,y,z ما خاص هستند !! نه این که کوشیتون غلطه ها . نه ! منظورم اینه که ما باید از این خاص بودن سه عددمون استفاده لازمه رو ببریم . ثانیا نتیجه کوشی شما زیر رادیکال بردن x,y,z هستش . حالا اگه دقت کنید در هر کدوم a,b,c زیر رادیکال هستند . حالا یه رادیکال هم بیاد روش میشه فرجه 4 که هم بد ریخت میشه و هم کارمون رو بسیار سخت میکنه :3:
 

m-saghaei

New Member
ارسال ها
338
لایک ها
258
امتیاز
0
پاسخ : ماراتن نامساوي

سلام
1- m و n دو عدد طبیعین که اختلافشون بیشتر از یکه. و m+n|4mn+1 ثابت کنید ب.م.م 2n-1 و 2m+1 برابر با یکه.
.
.
.
ببخشید حواسم نبود گذاشتم تو نامساوی! پوزش میطلبم!
 
آخرین ویرایش توسط مدیر

m-saghaei

New Member
ارسال ها
338
لایک ها
258
امتیاز
0
پاسخ : ماراتن نامساوي

یه سوال داشتم:
اگر
و
نشان دهید:

 

m-saghaei

New Member
ارسال ها
338
لایک ها
258
امتیاز
0
پاسخ : ماراتن نامساوي

اینم یه سوال قشنگ:

,
ثابت کنید:
 
بالا